Simplifying the radicals

Algebra Level 3

The value of S S is an integer I I . Enter the value of I I .

S = 41 + 29 2 5 29 2 41 5 S=\sqrt[5] {41+29\sqrt {2}}-\sqrt[5] {29\sqrt {2}-41}

You may not use a calculator.

Assumptions \textbf{Assumptions}

Consider a 5 \sqrt[5] {a} to be the principal fifth root of a a R a \ \forall a \in \mathbb{R} .


The answer is 2.

This section requires Javascript.
You are seeing this because something didn't load right. We suggest you, (a) try refreshing the page, (b) enabling javascript if it is disabled on your browser and, finally, (c) loading the non-javascript version of this page . We're sorry about the hassle.

4 solutions

Karim Mohamed
Aug 15, 2014

s = 41 + 29 2 5 29 2 41 5 s=\sqrt[5]{41+29*\sqrt{2}}-\sqrt[5]{29*\sqrt{2}-41}

In the first term in R.H.S multiply and divide by 41 29 2 5 \sqrt[5]{41-29\sqrt{2}}

s = 4 1 2 2 2 9 2 5 41 29 2 5 + 41 29 2 5 s=\frac{\sqrt[5]{41^{2}-2*29^{2}}}{\sqrt[5]{41-29\sqrt{2}}}+\sqrt[5]{41-29\sqrt{2}}

4 1 2 2 2 9 2 = 1 41^{2}-2*29^{2} = -1 this can be easily calculated without a calculator.

s = 1 41 29 2 5 + 41 29 2 5 s= \frac{-1}{\sqrt[5]{41-29\sqrt{2}}}+ \sqrt[5]{41-29\sqrt{2}}

let a = 41 29 2 5 a = \sqrt[5]{41-29\sqrt{2}}

s = 1 a + a s=\frac{-1}{a} +a

a 2 s a 1 = 0 a^{2}-s*a-1=0

so, a = s ± s 2 + 4 2 a=\frac{s\pm\sqrt{s^{2}+4}}{2}

since, a = 41 29 2 5 < 0 a = \sqrt[5]{41-29\sqrt{2}} < 0

so, 41 29 2 5 = s s 2 + 4 2 \sqrt[5]{41-29\sqrt{2}} =\frac{s-\sqrt{s^{2}+4}}{2}

By taking a power of 5 to both sides

32 ( 41 29 2 ) = ( s s 2 + 4 ) 5 32*(41-29\sqrt{2} )=(s-\sqrt{s^{2}+4} )^{5}

We can expand the R.H.S using the binomial theorem

R.H.S = s 5 5 s 4 s 2 + 4 + 10 s 3 s 2 + 4 10 s 2 ( s 2 + 4 ) s 2 + 4 + 5 s s 2 + 4 ( s 2 + 4 ) 2 s 2 + 4 =s^{5} -5*s^{4} *\sqrt{s^{2}+4} +10*s^{3} *\sqrt{s^{2}+4} -10*s^{2} *(s^{2} +4)*\sqrt{s^{2}+4} +5*s*\sqrt{s^{2}+4} -(s^{2} +4)^{2} *\sqrt{s^{2}+4}

By looking at the L.H.S we see it consists of two terms, one of them containing 2 \sqrt{2} so the term free from 2 \sqrt{2} should equal the summation of the terms free of s 2 + 4 \sqrt{s^{2}+4} in the R.H.S

so by taking the terms free of s 2 + 4 \sqrt{s^{2}+4} and simplifying then we get,

16 s 5 + 80 s 3 + 80 s = 32 41 16*s^{5} +80*s^{3} +80*s=32*41

so,

s ( s 4 + 4 s 2 + 4 ) = 2 41 s*(s^{4} +4*s^{2} +4)=2*41

2 and 41 are prime numbers.

so in the L.H.S one term should be 2 and the other should be 41.

obviously s = 2 s=2 , and by substitution of s = 2 s=2 in ( s 4 + 4 s 2 + 4 ) (s^{4} +4*s^{2} +4) we get 41.

so

s = 2 s=2

S Sen
Aug 16, 2014

Wow..... some awesome solutions here.... My solution is a little primitive... We know 49*sqrt(2) ~ 41.006 (pen and paper) ...

So the First term is (81.006)^(1/5) and the second term is (0.006)^(1/5) Observing 2^5=32 and 3^5=243 , the first term must lie between (2,3).

For the second term , we observe that 0.5^5~0.031 (paper and pen)... Therefore, the second expression is a value between (0,0.5) .

Finally, the maximum and minimum bounds for the expression has to be 3 and 1.5 . So speculative range is (1.5,3) ... (open interval ---- as second term will yield a positive value).... Therefore, only integer value possible is 2.

First note that S = 41 + 29 2 5 + 41 29 2 5 S=\sqrt[5]{41+29\sqrt{2}}+\sqrt[5]{41-29\sqrt{2}} (after rewriting a little) is the sum of the real fifth roots of the solutions of some quadratic polynomial P ( z ) P(z) . So, let z = 41 + 29 2 z=41+29\sqrt{2} . We will find the minimal polynomial, which also contains the conjugate z = 41 29 2 z=41-29\sqrt{2} :

z = 41 + 29 2 ( z 41 ) 2 = 1682 z 2 82 z 1 = 0 z=41+29\sqrt{2} \Longrightarrow (z-41)^2=1682 \Longrightarrow z^2-82z-1=0

So, P ( z ) = z 2 82 z 1 P(z)=z^2-82z-1 .

Now, let's save that result and try to find a general solution for the polynomial P ( x ) = x 5 + 5 a x 3 + 5 a 2 x + b = 0 P(x)=x^5+5ax^3+5a^2x+b=0 . The identity ( u + v ) 5 5 u v ( u + v ) 3 + 5 ( u v ) 2 ( u + v ) ( u 5 + v 5 ) = 0 (u+v)^5-5uv(u+v)^3+5(uv)^2(u+v)-(u^5+v^5)=0 will help us because we can compare coefficient by coefficient with the polynomial and obtain:

x = u + v x=u+v

5 u v = 5 a u v = a u 5 v 5 = a 5 -5uv=5a \Longrightarrow uv=-a \Longrightarrow u^5v^5=-a^5

( u 5 + v 5 ) = b u 5 + v 5 = b -(u^5+v^5)=b \Longrightarrow u^5+v^5=-b

Observe that by Vieta's formulas, u 5 u^5 and v 5 v^5 are the solutions for some quadratic polynomial P ( z ) P(z) . But wait, we already know that polynomial!. By the first equation, clearly S S is a root of P ( x ) P(x) . So let's compare again:

u 5 + v 5 = ( 82 ) b = 82 u^5+v^5=-(-82) \Longrightarrow b=-82

u 5 v 5 = 1 a 5 = 1 a = 1 u^5v^5=-1 \Longrightarrow a^5=1 \Longrightarrow a=1

Substitute a a and b b in the polynomial P ( x ) P(x) , which would be the minimal polynomial of S S :

P ( x ) = x 5 + 5 x 3 + 5 x 82 = 0 P(x)=x^5+5x^3+5x-82=0

By the rational root test, it factors as:

P ( x ) = ( x 2 ) ( x 4 + 2 x 3 + 9 x 2 + 18 x + 41 ) P(x)=(x-2)(x^4+2x^3+9x^2+18x+41)

We don't need to check that the fourth degree polynomial doesn't have real roots, because the expressions inside the fifth roots is a real number, and there are other four complex combinations that actually are the remaining roots. (They are u 5 w k + v 5 w 5 k \sqrt[5]{u}w^k+\sqrt[5]{v}w^{5-k} for k = 1 , 2 , 3 , 4 k=1,2,3,4 , where w w is any primitive fifth root of unity).

Hence, x = 2 x=2 and S = 2 \boxed{S=2} .

Panya Chunnanonda
Sep 12, 2014

Because (( sqrt. 2)+ 1 )^5= (29* sqrt. 2)+ 41 and ((sqrt. 2)- 1)^5= (29* sqrt. 2)- 41 So, S= ((sqrt. 2)+ 1)- ((sqrt. 2)- 1) S= 1- (-1)= 2

0 pending reports

×

Problem Loading...

Note Loading...

Set Loading...